Critère d'Eisenstein

Bonjour
J'étudie le critère d'Eisenstein.

Soit $A=a_nX^n+ \dots + a_1X+a_0 \in \Z[X]$ et $p$ un nombre premier.
On suppose que :
(i) $p$ ne divise pas $a_n$ ;
(ii) $p$ divise $a_0, a_1, \dots, a_{n-1}$;
(iii) $p^2$ ne divise pas $a_0$.
Alors $A$ est irréductible dans $\Q[X]$.

J'ai montré que : si $A\in \Z[X]$ est irréductible dans $\Z[X]$, il est irréductible dans $\Q[X]$.

Je raisonne par l'absurde et suppose $A$ réductible dans $\Q[X]$. $A$ est alors réductible dans $\Z[X]$.
Donc il existe $B$ et $C$ dans $\Z[X]$, tels que \[A=BC \quad \mbox{avec} \quad q=\mbox{deg}(B)\geq 1 \quad \mbox{et} \quad r=\mbox{deg}(C)\geq 1.

\] Écrivons $\displaystyle B= \sum_{k=0}^q b_kX^k,\quad C= \sum_{k=0}^r c_kX^k$.
Je montre que les polynômes $\overline{B}$ et $\overline{C}$ sont de degrés respectifs $q$ et $r$.
C'est ce morceau de mon corrigé que je ne comprends pas.
corrigé a écrit:
Par unicité de la décomposition en irréductibles dans $\Z/p\Z[X]$, on a : \[\overline{B}=\overline{b_q}X^q\ \quad \mbox{et} \quad \overline{C}=\overline{c_r}X^r.\]
On a : $\overline{a_n}X^n=\overline{A}=\overline{B}\;\overline{C}$ mais rien n'indique que cette décomposition est une décomposition en irréductibles donc je ne comprends pas bien l'argument.

Ou quelque chose m'échappe ?

Réponses

  • Bonjour,
    cela vient tout simplement du fait que $\overline{A}(X)=\overline{a_n}X^n$ dans $\Z/p\Z[x]$, le polnôme $\overline{B}$ est donc un diviseur de $\overline{A}$... Après c'est vrai que l'argument "unicité de la décomposition dans $\Z/p\Z[X]$" est un peu ambigu.
    A+
    F.
  • OK, je comprends mieux. Merci malavita.

    L'argument de l'unicité de la décomposition est seulement "un peu ambigu" ? Il n'est pas juste non ?
  • cuba a écrit:
    J'ai montré que : si $A\in \Z[X]$ est irréductible dans $\Z[X]$, il est irréductible dans $\Q[X]$.

    En l'état c'est faux. À moins que tu ne l'énonces pour le $A$ de ton énoncé, pour lequel ça va fonctionner.
  • Bonjour Poirot,

    Tu crées un doute chez moi ! Je n'ai pas utilisé de "forme particulière pour $A$".

    J'ai procédé en m'inspirant de Gourdon :
    a écrit:
    On suppose $A\in \Z[X]$ irréductible dans $\Z[X]$.
    Soient $P, Q \in \Q[X]$ tels que $A=PQ$. Soient $\alpha, \beta \in \N^*$ tels que $P_1=\alpha P$ et $Q_1 = \beta Q \in \Z[X]$.
    On a $\alpha \beta A = P_1Q_1$ d'où: $\alpha \beta c(A)=c(P_1)c(Q_1)$ où $c()$ désigne le contenu.
    Posons $P_2=\dfrac{P_1}{c(P_1)}$ et $Q_2=\dfrac{Q_1}{c(Q_1)}$. Ces polynômes sont à coefficients entiers. Par ailleurs, \[\alpha \beta A = c(P_1)c(Q_1)P_2Q_2=\alpha \beta c(A) P_2Q_2.\]
    Si $P_3=c(A)P_2$, on a donc $A=P_3Q_2$ avec $P_3,Q_2 \in \Z[X]$.
    Comme $A$ est irréductible dans $\Z[X]$, on a nécessairement \[\mbox{deg}(P_3)=0 \quad \mbox{ou} \quad \mbox{deg}(Q_2)=0.\]
    Donc $\mbox{deg}(P)=0$ ou $\mbox{deg}(Q)=0$, ce qui prouve que $A$ est bien irréductible dans $\Q[X]$.
  • Le polynôme $2$ est irréductible dans $\mathbb Z[X]$, mais pas dans $\mathbb Q[X]$.
  • Par définition, un polynôme irréductible n'est pas de degré supérieur ou égal à 1 ?
  • Re,

    c'est justement une des "feintes" de l'arithmétique sur $\Z$, le polynôme constant égal à $6$ est irréductible sur $\Q$ mais pas sur $\Z$, car $6=2 \times 3$ ...
    Pour qu'un polynôme irréductible sur $\Q$ le soit aussi sur $\Z$, il faut ajouter une condition sur les coefficients... de mémoire je dirais que le pgcd de ses coefficients doit être égal à 1 (à confirmer).
    Par rapport à l’ambiguïté que je soulevais, je voulais dire que la décomposition $\overline{A}=\overline{B}\times\overline{C}$ n'est pas la décomposition en facteurs irréductibles.

    Bonne journée.
    F.
  • @malavita : $6$ n'est irréductible ni dans $\mathbb Z[X]$ (puisqu'il s'écrit $2 \times 3$ avec $2$ et $3$ non inversibles), ni dans $\mathbb Q[X]$ (car inversible).

    @cuba : Un élément irréductible d'un anneau est un élément non inversible tel que si celui-ci s'écrit comme produit de deux éléments de l'anneau, alors l'un des deux est inversible.
  • Ok c'est bon pour moi. Un grand merci à tous les deux.
  • Et oui les unités ne sont pas irréductibles (td)

    Bonne soirée
Connectez-vous ou Inscrivez-vous pour répondre.